The Student Room Group

The Proof is Trivial!

Scroll to see replies

Is applied mathematics allowed on here?
Reply 3041
Original post by rayquaza17
Problem 494 ***

Evaluate:
0cos(ax)1+x2dx\displaystyle \int_{0}^{\infty }\frac{cos(ax)}{1+x^2}dx for a0a\geq 0.

Sorry if it's been asked before.


Spoiler

Original post by SParm

Spoiler



Spoiler




Original post by Arieisit
Is applied mathematics allowed on here?

Yeah. Maybe if it's some easy-ish applied maths I might be able to actually answer a question. :redface:
Reply 3043
Original post by rayquaza17

Spoiler





Yeah. Maybe if it's some easy-ish applied maths I might be able to actually answer a question. :redface:


Yeah sorry very possible I messed up by a factor of a half, won't be the first or last time.
Me looking through this thread
Original post by ThatPerson
From IMO 2013 Q1.

Problem 492**

Prove that for any pair of positive integers kk and n n, there exist k k positive integers m1,m2,...,mk m_1,m_2,...,m_k (not necessarily different) such that

1+2k1n=(1+1m1)(1+1m2)...(1+1mk) 1 + \dfrac{2^k - 1}{n} = \left(1 + \dfrac{1}{m_1}\right)\left(1 + \dfrac{1}{m_2}\right)...\left(1 + \dfrac{1}{m_k}\right)


After 15 hours, I have finally solved it... Finally...
Now I can do some revision haha :smile:
Original post by Renzhi10122
After 15 hours, I have finally solved it... Finally...
Now I can do some revision haha :smile:


Nice. Although I'm not sure if I should feel somewhat guilty for distracting you :tongue:.
Original post by ThatPerson
Nice. Although I'm not sure if I should feel somewhat guilty for distracting you :tongue:.


Haha, around 13 of those 15 hours were spent on the question a few weeks ago, so no worries.
Original post by Renzhi10122
After 15 hours, I have finally solved it... Finally...
Now I can do some revision haha :smile:


Proof or gtfo
Original post by HeavisideDelts
Proof or gtfo


Fine then (warning, slightly long and inelegant)

Spoiler

Original post by Renzhi10122
Fine then (warning, slightly long and inelegant)

Spoiler


Niiiiice. Basically by induction on the binary expansion, I suppose, but you've gone and done a constructive proof.
Original post by Smaug123
Niiiiice. Basically by induction on the binary expansion, I suppose, but you've gone and done a constructive proof.


Yeah, induction would probably have made it easier, and less time consuming
Original post by Mladenov



There are hundreds of techniques when it comes to functional equations. It is annoying that there are no books on functional eqs; I could think of 1-2 really good for olympiad problems.


.


plz can you share them
the methods and techniques
im aweful
Original post by demigawdz
plz can you share them
the methods and techniques
im aweful


At and below BMO2 level, substitution is the main one. Sub in some values, see if you get any good relations, see if you can get any values for some f(x). At a higher level, check for injectivity and surjectivity (you may have to look on wikipedia if you don't know what these are). Functional equations also get more algebraic as they get harder, so then a set method doesn't really apply. Someone probably has better advice than this.
Original post by Renzhi10122
At and below BMO2 level, substitution is the main one. Sub in some values, see if you get any good relations, see if you can get any values for some f(x). At a higher level, check for injectivity and surjectivity (you may have to look on wikipedia if you don't know what these are). Functional equations also get more algebraic as they get harder, so then a set method doesn't really apply. Someone probably has better advice than this.

do you know whether for q 5 0 counts as being in the set of ''non negative integers'' as the positive integer sign means positive lol (which doesnt include 0)/
http://www.bmoc.maths.org/home/bmo1-2002.pdf
Thank you.
Original post by demigawdz
do you know whether for q 5 0 counts as being in the set of ''non negative integers'' as the positive integer sign means positive lol (which doesnt include 0)/
http://www.bmoc.maths.org/home/bmo1-2002.pdf
Thank you.


I would say 0 is in the set of non-negative integers.

But Wolfram Alpha disagrees with what they define as the non-negative integers: http://mathworld.wolfram.com/NonnegativeInteger.html
(edited 8 years ago)
Original post by demigawdz
do you know whether for q 5 0 counts as being in the set of ''non negative integers'' as the positive integer sign means positive lol (which doesnt include 0)/
http://www.bmoc.maths.org/home/bmo1-2002.pdf
Thank you.


0 is indeed in the set of non-negative integers, although I always thought that Z+ was the set of positive integers.
Original post by Renzhi10122
0 is indeed in the set of non-negative integers, although I always thought that Z+ was the set of positive integers.


precisely why i am confuzzled by this
Original post by demigawdz
precisely why i am confuzzled by this


In the question, I guess its the set of non-negative integers then.
Original post by Renzhi10122
In the question, I guess its the set of non-negative integers then.


im so aweful at maths i cannot even do this one.
lemme guess its probably an easy one

Quick Reply

Latest